Adv Med-Surg CV

Réussis tes devoirs et examens dès maintenant avec Quizwiz!

ANS: D Sinus rhythm with PVCs has an underlying regular sinus rhythm with ventricular depolarization that sometimes precede atrial depolarization. Ventricular tachycardia and ventricular fibrillation rhythms would not have sinus beats present. Premature atrial contractions are atrial contractions initiated from another region of the atria before the sinus node initiates atrial depolarization.

A nurse assesses a clients electrocardiogram (ECG) and observes the reading shown below: How should the nurse document this clients ECG strip? a. Ventricular tachycardia b. Ventricular fibrillation c. Sinus rhythm with premature atrial contractions (PACs) d. Sinus rhythm with premature ventricular contractions (PVCs)

The nurse should evaluate that defibrillation of a client was most successful if which observation was made? 1. Arousable, sinus rhythm, blood pressure (BP) 116/72 mm Hg 2. Nonarousable, sinus rhythm, BP 88/60 mm Hg 3. Arousable, marked bradycardia, BP 86/54 mm Hg 4. Nonarousable, supraventricular tachycardia, BP 122/60 mm Hg

ANS: 1 After defibrillation, the client requires continuous monitoring of electrocardiographic rhythm, hemodynamic status, and neurological status. Respiratory and metabolic acidosis develop during ventricular fibrillation because of lack of respiration and cardiac output. These can cause cerebral and cardiopulmonary complications. Arousable status, adequate BP, and a sinus rhythm indicate successful response to defibrillation.

The nurse is assessing the neurovascular status of a client who returned to the surgical nursing unit 4 hours ago after undergoing aortoiliac bypass graft. The affected leg is warm, and the nurse notes redness and edema. The pedal pulse is palpable. How should the nurse interpret the client's neurovascular status? 1. The neurovascular status is normal because of increased blood flow through the leg. 2. The neurovascular status is moderately impaired, and the surgeon should be called. 3. The neurovascular status is slightly deteriorating and should be monitored for another hour. 4. The neurovascular status shows adequate arterial flow, but venous complications are arising.

ANS: 1 An expected outcome of aortoiliac bypass graft surgery is warmth, redness, and edema in the surgical extremity because of increased blood flow. The remaining options are incorrect interpretations.

The nurse is watching the cardiac monitor, and a client's rhythm suddenly changes. There are no P waves; instead, there are fibrillatory waves before each QRS complex. How should the nurse interpret the client's heartrhythm? 1. Atrial fibrillation 2. Sinus tachycardia 3. Ventricular fibrillation 4. Ventricular tachycardia

ANS: 1 Atrial fibrillation is characterized by a loss of P waves and fibrillatory waves before each QRS complex. The atria quiver, which can lead to thrombus formation.

A nurse assesses a client who is recovering after a left-sided cardiac catheterization. Which assessment finding requires immediate intervention? a. Urinary output less than intake b. Bruising at the insertion site c. Slurred speech and confusion d. Discomfort in the left leg

ANS: C A left-sided cardiac catheterization specifically increases the risk for a cerebral vascular accident. A change in neurologic status needs to be acted on immediately. Discomfort and bruising are expected at the site. If intake decreases, a client can become dehydrated because of dye excretion. The second intervention would be to increase the clients fluid status. Neurologic changes would take priority.

A nurse is assessing clients who have intravenous therapy prescribed. Which assessment finding for a client with a peripherally inserted central catheter (PICC) requires immediate attention? a. The initial site dressing is 3 days old. b. The PICC was inserted 4 weeks ago. c. A securement device is absent. d. Upper extremity swelling is noted.

ANS: D Upper extremity swelling could indicate infiltration, and the PICC will need to be removed. The initial dressing over the PICC site should be changed within 24 hours. This does not require immediate attention, but the swelling does. The dwell time for PICCs can be months or even years. Securement devices are being used more often now to secure the catheter in place and prevent complications such as phlebitis and infiltration. The IV should have one, but this does not take priority over the client whose arm is swollen.

A nurse prepares to flush a peripherally inserted central catheter (PICC) line with 50 units of heparin. The pharmacy supplies a multi-dose vial of heparin with a concentration of 100 units/mL. Which of the syringes should the nurse use to draw up and administer the heparin?

Always use a 10-mL syringe when flushing PICC lines because a smaller syringe creates higher pressure, which could rupture the lumen of the PICC.

The nurse working in the emergency department knows that which factors are commonly related to aneurysm formation? (Select all that apply.) a. Atherosclerosis b. Down syndrome c. Frequent heartburn d. History of hypertension e. History of smoking ANS: A, D, E

Atherosclerosis, hypertension, hyperlipidemia, and smoking are the most common related factors. Down syndrome and heartburn have no relation to aneurysm formation.

A nurse assesses a client who has aortic regurgitation. In which location should the nurse auscultate to best hear a cardiac murmur related to aortic regurgitation?

In the second intercostal space just to the right of the sternum (aortic valve)

The nurse is watching the cardiac monitor and notices that a client's rhythm suddenly changes. There are no P waves, the QRS complexes are wide, and the ventricular rate is regular but more than 140 beats per minute. The nurse determines that the client is experiencing which dysrhythmia? 1. Sinus tachycardia 2. Ventricular fibrillation 3. Ventricular tachycardia 4. Premature ventricular contractions

ANS: 3 Ventricular tachycardia is characterized by the absence of P waves, wide QRS complexes (longer than 0.12 seconds), and typically a rate between 140 and 180 impulses per minute. The rhythm is regular.

A client is admitted to the emergency department with chest pain that is consistent with myocardial infarction based on elevated troponin levels. Heart sounds are normal. The nurse should alert the primary health care provider d/t vital sign changes (increased HR, increased RR, and decrease BP) and client assessment are most consistent with which complication?

ANS: 1 Cardiogenic shock occurs with severe damage (more than 40%) to the left ventricle. Classic signs include hypotension; a rapid pulse that becomes weaker; decreased urine output; and cool, clammy skin. Respiratory rate increases as the body develops metabolic acidosis from shock. Cardiac tamponade is accompanied by distant, muffled heart sounds and prominent neck vessels. Pulmonary embolism presents suddenly with severe dyspnea accompanying the chest pain. Dissecting aortic aneurysms usually are accompanied by back pain.

A client's electrocardiogram strip shows atrial and ventricular rates of 110 bpm. The PR interval is 0.14 seconds, the QRS complex measures 0.08 seconds, and the PP and RR intervals are regular. How should the nurse interpret this rhythm? 1. Sinus tachycardia 2. Sinus bradycardia 3. Sinus dysrhythmia 4. Normal sinus rhythm

ANS: 1 Sinus tachycardia has the characteristics of normal sinus rhythm, including a regular PP interval and normal-width PR and QRS intervals; however, the rate is the differentiating factor. In sinus tachycardia, the atrial and ventricular rates are greater than 100 beats per minute.

A client has frequent bursts of ventricular tachycardia on the cardiac monitor. What should the nurse be most concerned about with this dysrhythmia? 1. It can develop into ventricular fibrillation at any time. 2. It is almost impossible to convert to a normal rhythm. 3. It is uncomfortable for the client, giving a sense of impending doom. 4. It produces a high cardiac output with cerebral and myocardial ischemia.

ANS: 1 Ventricular tachycardia is a life-threatening dysrhythmia that results from an irritable ectopic focus that takes over as the pacemaker for the heart. Ventricular tachycardia can deteriorate into ventricular fibrillation at any time. Clients frequently experience a feeling of impending doom. The low cardiac output that results can lead quickly to cerebral and myocardial ischemia. Ventricular tachycardia is treated with antidysrhythmic medications, cardioversion (if the client is awake), or defibrillation (loss of consciousness).

The client has developed atrial fibrillation, with a ventricular rate of 150 bpm. The nurse should assess the client for which associated signs and/or symptoms? Select all that apply. 1. Syncope 2. Dizziness 3. Palpitations 4. Hypertension 5. Flat neck veins

ANS: 1, 2, 3 The client with uncontrolled atrial fibrillation with a ventricular rate more than 100 beats per minute is at risk for low cardiac output because of loss of atrial kick. The nurse assesses the client for palpitations, chest pain or discomfort, hypotension, pulse deficit, fatigue, weakness, dizziness, syncope, shortness of breath, and distended neck veins. Hypertension and flat neck veins are not associated with the loss of cardiac output.

The nurse in a medical unit is caring for a client with heart failure. The client suddenly develops extreme dyspnea, tachycardia, and lung crackles. The nurse immediately asks another nurse to contact the primary health care provider and prepares to implement which priority interventions? (SAA). 1. Administering oxygen 2. Inserting a Foley catheter 3. Administering furosemide 4. Administering morphine sulfate intravenously 5. Transporting the client to the coronary care unit 6. Placing the client in a low-Fowler's side-lying position

ANS: 1, 2, 3, 4 Extreme dyspnea, tachycardia, and lung crackles in a client with HF indicate pulmonary edema, a life-threatening event. In pulmonary edema, the left ventricle fails to eject sufficient blood, and pressure increases in the lungs d/t accumulated blood. Oxygen is always prescribed, and the client is placed in a high-Fowler's position to ease the work of breathing. Furosemide, a rapid-acting diuretic, will eliminate accumulated fluid. A Foley catheter is inserted to measure output accurately. Intravenously administered morphine sulfate reduces venous return (preload), decreases anxiety, and also reduces the work of breathing. Transporting the client to the coronary care unit is not a priority intervention. In fact, this may not be necessary at all if the client's response to treatment is successful.

The nurse is monitoring a client for adverse effects of medications. Which findings are characteristic of adverse effects of hydrochlorothiazide? (SAA) 1. Sulfa allergy 2. Osteoporosis 3. Hypokalemia 4. Hypouricemia 5. Hyperglycemia 6. Hypercalcemia

ANS: 1, 3, 5, 6 Thiazide diuretics such as hydrochlorothiazide are sulfa-based medications, and a client with a sulfa allergy is at risk for an allergic reaction. Also, clients are at risk for hypokalemia, hyperglycemia, hypercalcemia, hyperlipidemia, and hyperuricemia.

A client receiving thrombolytic therapy with a continuous infusion of a-TP suddenly becomes extremely anxious and reports itching. The nurse hears stridor and notes generalized urticaria and hypotension. Which interventions should the nurse anticipate? (SSA) 1. Stop the infusion. 2. Raise the head of the bed. 3. Administer protamine sulfate. 4. Administer diphenhydramine. 5. Call for the Rapid Response Team (RRT).

ANS: 1, 4, 5 The client is experiencing an anaphylactic reaction. Therefore, the priority action is to stop the infusion and notify the RRT. The client may be treated with antihistamines. Raising the head of the bed would not be helpful, as that may exacerbate the hypotension. Protamine sulfate is the antidote for heparin, so it is not useful for a client receiving a-TP.

The nurse is monitoring a client with hypertension who is taking propranolol. Which assessment finding indicates a potential adverse complication associated with this medication? 1. Report of infrequent insomnia 2. Development of expiratory wheezes 3. A baseline BP of 150/80, followed by a BP of 138/72 after 2 doses of the medication 4. A baseline resting heart rate of 88 beats per minute followed by a resting HR of 72 bpm after 2 doses of the medication

ANS: 2 Audible expiratory wheezes may indicate a serious adverse reaction, bronchospasm. Beta blockers may induce this reaction, particularly in clients with chronic obstructive pulmonary disease or asthma. Normal decreases in blood pressure and heart rate are expected. Insomnia is a frequent mild side effect and should be monitored.

A nurse is teaching a client with premature ectopic beats. Which education should the nurse include in this clients teaching? (Select all that apply.) a. Smoking cessation b. Stress reduction and management c. Avoiding vagal stimulation d. Adverse effects of medications e. Foods high in potassium

ANS: A, B, D A client who has premature beats or ectopic rhythms should be taught to stop smoking, manage stress, take medications as prescribed, and report adverse effects of medications. Clients with premature beats are not at risk for vasovagal attacks or potassium imbalances.

The nurse administered intravenous bumetanide to a client being treated for HF. Which outcome indicates that the medication has achieved the expected effect? 1. Cough becomes productive of frothy pink sputum. 2. Urine output increases from 10 mL/hr to greater than 50 mL hourly. 3. The serum potassium level changes from 3.8 to 3.1 mEq/L (3.8 to 3.1 mmol/L). 4. B-type natriuretic peptide (BNP) factor increases from 200 to 262 ng/mL (200 to 262 mcg/L).

ANS: 2 Bumetanide is a diuretic and expected to increase urine output, decrease crackles, and decrease weight. Potassium loss is a side effect rather than an expected effect of the diuretic. Frothy pink sputum indicates progression to pulmonary edema. A BNP greater than 100 pg/mL indicates HF; thus, a rise from a previous level indicates worsening of the condition.

A client with myocardial infarction is developing cardiogenic shock. What condition should the nurse carefully assess the client for? 1. Pulsus paradoxus 2. Ventricular dysrhythmias 3. Rising diastolic blood pressure 4. Falling central venous pressure

ANS: 2 Dysrhythmias commonly occur as a result of decreased oxygenation and severe damage to greater than 40% of the myocardium. Classic signs of cardiogenic shock as they relate to myocardial ischemia include low blood pressure & tachycardia. The central venous pressure would rise as the backward effects of the severe left ventricular failure became apparent. Pulsus paradoxus is a finding associated with cardiac tamponade.

The nurse is caring for a client who has just had implantation of an automatic internal cardioverter-defibrillator. The nurse should assess which item based on priority? 1. Anxiety level of the client and family 2. Activation status and settings of the device 3. Presence of a MedicAlert card for the client to carry 4. Knowledge of restrictions on postdischarge physical activity

ANS: 2 Nursing responsibilities after cardioversion include maintenance first of a patent airway, and then oxygen administration, assessment of vital signs and level of consciousness, and dysrhythmia detection.

The nurse is evaluating a client's response to cardioversion. Which assessment would be the priority? 1. Blood pressure 2. Airway patency 3. Oxygen flow rate 4. Level of consciousness

ANS: 2 Nursing responsibilities after cardioversion include maintenance first of a patent airway, and then oxygen administration, assessment of vital signs and level of consciousness, and dysrhythmia detection.

A client with myocardial infarction suddenly becomes tachycardic, shows signs of air hunger, and begins coughing frothy, pink-tinged sputum. Which finding would the nurse anticipate when auscultating the client's breath sounds? 1. Stridor 2. Crackles 3. Scattered rhonchi 4. Diminished breath sounds

ANS: 2 Pulmonary edema is characterized by extreme breathlessness, dyspnea, air hunger, and the production of frothy, pink-tinged sputum. Auscultation of the lungs reveals crackles. Rhonchi and diminished breath sounds are not associated with pulmonary edema. Stridor is a crowing sound associated with laryngospasm or edema of the upper airway.

A client is wearing a continuous cardiac monitor, which begins to sound its alarm. The nurse sees no electrocardiographic complexes on the screen. Which is the priority nursing action? 1. Call a code. 2. Check the client's status. 3. Call the health care provider. 4. Document the lack of complexes.

ANS: 2 Sudden loss of electrocardiographic complexes indicates ventricular asystole or possibly electrode displacement. Accurate assessment of the client is necessary to determine the cause and identify the appropriate intervention. The remaining options are secondary to client assessment.

Intravenous heparin therapy is prescribed for a client with atrial fibrillation. While implementing this prescription, the nurse ensures that which medication is available on the nursing unit? 1. Vitamin K 2. Protamine sulfate 3. Potassium chloride 4. Aminocaproic acid

ANS: 2 The antidote to heparin is protamine sulfate; it should be readily available for use if excessive bleeding or hemorrhage should occur.

A client who had cardiac surgery 24 hours ago has had a urine output averaging 20 mL/hr for 2 hours. The client received a single bolus of 500 mL of intravenous fluid. Urine output for the subsequent hour was 25 mL. Daily laboratory results indicate that the blood urea nitrogen level is 45 mg/dL (16 mmol/L) and the serum creatinine level is 2.2 mg/dL (194 mcmol/L). On the basis of these findings, the nurse would anticipate that the client is at risk for which problem? 1. Hypovolemia 2. Acute kidney injury 3. Glomerulonephritis 4. Urinary tract infection

ANS: 2 The client who undergoes cardiac surgery is at risk for renal injury from poor perfusion, hemolysis, low cardiac output, or vasopressor medication therapy. Renal injury is signaled by decreased urine output and increased BUN (10-20) and creatinine levels (M: 0.6 to 1.2; F: 0.5 to 1.1 mg/dL) medications to increase renal perfusion and possibly could need peritoneal dialysis or hemodialysis. No data in the question indicate the presence of hypovolemia, glomerulonephritis, or urinary tract infection.

The nurse is caring for a client who has just had implantation of an automatic internal cardioverter-defibrillator. The nurse should assess which item based on priority? 1. Anxiety level of the client and family 2. Activation status and settings of the device 3. Presence of a MedicAlert card for the client to carry 4. Knowledge of restrictions on postdischarge physical activity

ANS: 2 The nurse needs to assess device settings, similar to after insertion of a permanent pacemaker. Specifically, the nurse needs to know whether the device is activated, the heart rate cutoff above which it will fire, and the number of shocks it is programmed to deliver. The remaining options are also nursing interventions but are not the priority.

A client is diagnosed with an ST segment elevation myocardial infarction (STEMI) and is receiving a tissue plasminogen activator, alteplase. Which action is a priority nursing intervention? 1. Monitor for kidney failure. 2. Monitor psychosocial status. 3. Monitor for signs of bleeding. 4. Have heparin sodium available.

ANS: 3 Tissue plasminogen activator is a thrombolytic. Hemorrhage is a complication of any type of thrombolytic medication. The client is monitored for bleeding. Monitoring for renal failure and monitoring the client's psychosocial status are important but are not the most critical interventions. Heparin may be administered after thrombolytic therapy, but the question is not asking about follow- up medications.

A nurse assesses a client with mitral valve stenosis. What clinical manifestation should alert the nurse to the possibility that the clients stenosis has progressed? a. Oxygen saturation of 92% b. Dyspnea on exertion c. Muted systolic murmur d. Upper extremity weakness

ANS: B Dyspnea on exertion develops as the mitral valvular orifice narrows and pressure in the lungs increases. The other manifestations do not relate to the progression of mitral valve stenosis.

A client with valvular heart disease who has a clot in the right atrium is receiving a heparin sodium infusion at 1000 units/hr and warfarin sodium 7.5 mg at 5:00 p.m. daily. The morning laboratory results are as follows: activated partial thromboplastin time (aPTT), 32 seconds; international normalized ratio (INR), 1.3. The nurse should take which action based on the client's laboratory results? 1. Collaborate with the primary health care provider (PHCP) to discontinue the heparin infusion and administer the warfarin sodium as prescribed. 2. Collaborate with the PHCP to obtain a prescription to increase the heparin infusion and continue the warfarin sodium as prescribed. 3. Collaborate with the PHCP to withhold the warfarin sodium since the client is receiving a heparin infusion and the aPTT is within the therapeutic range. 4. Collaborate with the PHCP to continue the heparin infusion at the same rate and to discuss use of dabigatran etexilate in place of warfarin sodium.

ANS: 2 When a client is receiving warfarin for clot prevention due to atrial fibrillation, an INR of 2 to 3 is appropriate for most clients. Until the INR has achieved a therapeutic range, the client should be maintained on a continuous heparin infusion with the aPTT ranging between 60 and 80 seconds. Therefore, the nurse should collaborate with the HCP to obtain a prescription to increase the heparin infusion and to administer the warfarin as prescribed.

The nurse is monitoring a client with heart failure who is taking digoxin. Which findings are characteristic of digoxin toxicity? (SSA) 1. Tremors 2. Diarrhea 3. Irritability 4. Blurred vision 5. Nausea and vomiting

ANS: 2, 4, 5 Digoxin is a cardiac glycoside. The risk of toxicity can occur with the use of this medication. Toxicity can lead to life-threatening events and the nurse needs to monitor the client closely for signs of toxicity. Early signs of toxicity include GI manifestations such as anorexia, nausea, vomiting, and diarrhea. Subsequent manifestations include headache; visual disturbances (diplopia, blurred vision, yellow-green halos, and photophobia); drowsiness; fatigue; and weakness. Cardiac rhythm abnormalities can also occur. The nurse also monitors the digoxin level. The optimal therapeutic range for digoxin is 0.5 to 2.0 ng/mL

A client with atrial fibrillation is receiving a continuous heparin infusion at 1000 units/hr. The nurse determines that the client is receiving the therapeutic effect based on which results? 1. Prothrombin time of 12.5 seconds 2. Activated partial thromboplastin time of 28 seconds 3. Activated partial thromboplastin time of 60 seconds 4. Activated partial thromboplastin time longer than 120 seconds

ANS: 3 Common activated partial thromboplastin time (aPTT) are 30 to 40 seconds. Because the aPTT should be 1.5 to 2.5 times the normal value, the client's aPTT would be considered therapeutic if it was 60 seconds. Prothrombin time assesses response to warfarin therapy.

The nurse is evaluating the condition of a client after pericardiocentesis performed to treat cardiac tamponade. Which observation would indicate that the procedure was effective? 1. Muffled heart sounds 2. Client reports dyspnea 3. A rise in blood pressure 4. Jugular venous distention

ANS: 3 Following pericardiocentesis, the client usually expresses immediate relief. Heart sounds are no longer muffled or distant and blood pressure increases. Distended neck veins are a sign of increased venous pressure, which occurs with cardiac tamponade.

The nurse is reviewing an electrocardiogram rhythm strip. The P waves and QRS are regular. The PR interval is 0.16 seconds, and QRS measure 0.06 seconds. The overall heart rate is 64 beats per minute. Which action should the nurse take? 1. Check vital signs. 2. Check laboratory test results. 3. Monitor for any rhythm change. 4. Notify the primary health care provider.

ANS: 3 Normal sinus rhythm is defined as a regular rhythm, with an overall rate of 60 to 100 beats per minute. The PR and QRS measurements are normal, measuring between 0.12 and 0.20 seconds and 0.04 and 0.10 seconds, respectively. There are no irregularities in this rhythm currently, so there is no immediate need to check vital signs or laboratory results, or to notify the primary health care provider. Therefore, the nurse would continue to monitor the client for any rhythm change.

A client is having frequent premature ventricular contractions. The nurse should place priority on assessment of which item? 1. Causative factors, such as caffeine 2. Sensation of fluttering or palpitations 3. Blood pressure and oxygen saturation 4. Precipitating factors, such as infection

ANS: 3 Premature ventricular contractions can cause hemodynamic compromise. Therefore, the priority is to monitor the blood pressure and oxygen saturation. The shortened ventricular filling time can lead to decreased cardiac output. The client may be asymptomatic or may feel palpitations. Premature ventricular contractions can be caused by cardiac disorders; states of hypoxemia; any number of physiological stressors, such as infection, illness, surgery, or trauma; and intake of caffeine, nicotine, or alcohol.

A client is being treated with procainamide for a cardiac dysrhythmia. Following intravenous administration of the medication, the client complains of dizziness. What intervention should the nurse take first? 1. Obtain a 12-lead electrocardiogram. 2. Check the client's fingerstick blood glucose level. 3. Auscultate the client's apical pulse and blood pressure. 4. Measure the QRS interval duration on the rhythm strip.

ANS: 3 Signs of toxicity from procainamide include confusion, dizziness, drowsiness, decreased urination, nausea, vomiting, and tachydysrhythmias. If the client complains of dizziness, the nurse should assess the vital signs first. Although measuring the QRS duration on the rhythm strip and obtaining a 12-lead EKG may be interventions, these would be done after the vital signs are taken. Dizziness directly following the procainamide indicates that the medication was the likely cause and should be addressed before assessing for other possible causes such as hypoglycemia.

The nurse should report which assessment finding to the PHCP before initiating thrombolytic therapy in a client with PE? 1. Adventitious breath sounds 2. Temperature of 99.4° F (37.4° C) orally 3. Blood pressure of 198/110 mm Hg 4. Respiratory rate of 28 breaths per minute

ANS: 3 Thrombolytic therapy is contraindicated in severe uncontrolled HTN d/t the risk of cerebral hemorrhage. Therefore, the nurse would report the results of the blood pressure to the PHCP before initiating therapy.

The nurse notes that a client with sinus rhythm has a premature ventricular contraction that falls on the T wave of the preceding beat. The client's rhythm suddenly changes to one with no P waves, no definable QRS and coarse wavy lines of varying amplitude. How should the nurse interpret this rhythm? 1. Asystole 2. Atrial fibrillation 3. Ventricular fibrillation 4. Ventricular tachycardia

ANS: 3 Ventricular fibrillation is characterized by irregular chaotic undulations of varying amplitudes. Ventricular fibrillation has no measurable rate and no visible P waves or QRS complexes and results from electrical chaos in the ventricles.

Prior to administering a client's daily dose of digoxin to tx HF the nurse reviews the client's laboratory data and notes the following results: serum calcium, 9.8 mg/dL; serum magnesium, 1.0 mEq/L; serum potassium, 4.1 mEq/L; serum creatinine, 0.9 mg/dL. Which result should alert the nurse that the client is at risk for digoxin toxicity? 1. Serum calcium level 2. Serum potassium level 3. Serum creatinine level 4. Serum magnesium level

ANS: 4 An increased risk of toxicity exists in clients with hypercalcemia, hypokalemia, hypomagnesemia, hypothyroidism, and impaired renal function. The calcium, creatinine, and potassium levels are all within normal limits. Magnesium is 1.8-2.6 mEq/L, and the results in the correct option are reflective of hypomagnesemia.

The nurse provides discharge instructions to a client with atrial fibrillation who is taking warfarin sodium. Which statement, by the client, reflects the need for further teaching? 1. "I will avoid alcohol consumption." 2. "I will take my pills every day at the same time." 3. "I have already called my family to pick up a MedicAlert bracelet." 4. "I will take coated aspirin for my headaches because it will coat my stomach."

ANS: 4 Aspirin-containing products need to be avoided when a client is taking this medication. Alcohol consumption should be avoided by a client taking warfarin sodium. Taking the prescribed medication at the same time each day increases client compliance. The MedicAlert bracelet provides health care personnel with emergency information.

The nurse provides instructions to the client about nicotinic acid prescribed for hyperlipidemia. Which statement by the client indicates understanding of the instructions? 1. "The medication should be taken with meals to decrease flushing." 2. "It is not necessary to avoid the use of alcohol when taking nicotinic acid." 3. "Clay-colored stools are a common side effect and should not be of concern." 4. "Ibuprofen taken 30 minutes before the nicotinic acid may decrease the flushing."

ANS: 4 Flushing is an adverse effect of this medication. ASA, NSAIDs, as prescribed, can be taken 30 minutes prior to taking the medication to decrease flushing. Alcohol consumption needs to be avoided because it will enhance this effect. The medication should be taken with meals to decrease GI upset; however, taking the medication with meals has no effect on the flushing. Clay-colored stools are a sign of hepatic dysfunction and should be reported to the primary health care provider (PHCP) immediately.

The home health care nurse is visiting a client with coronary artery disease with elevated triglyceride levels and a serum cholesterol level of 398 mg/dL. The client is taking cholestyramine, and the nurse teaches the client about the medication. Which statement by the client indicates the need for further teaching? 1. "Constipation and bloating might be a problem." 2. "I'll continue to watch my diet and reduce my fats." 3. "Walking a mile each day will help the whole process." 4. "I'll continue my nicotinic acid from the health food store."

ANS: 4 Nicotinic acid, even an over-the-counter form, should be avoided b/c it may lead to liver abnormalities. All lipid-lowering medications also can cause liver abnormalities, so a combination of nicotinic acid and cholestyramine resin needs to be avoided. Constipation and bloating are the 2 most common adverse effects. Walking and the reduction of fats in the diet are therapeutic measures to reduce cholesterol and triglyceride levels.

A client in ventricular fibrillation is about to be defibrillated. To convert this rhythm effectively, the monophasic defibrillator machine should be set at which energy level (in joules, J) for the first delivery? 1. 50 J 2. 120 J 3. 200 J 4. 360 J

ANS: 4 The energy level used for all defibrillation attempts with a monophasic defibrillator is 360 joules.

A client who is receiving digoxin daily has a serum potassium level of 3 mEq/L and reports anorexia. The health care provider prescribes a serum digoxin level to be done. The nurse checks the results and should recognize which level that is outside of the therapeutic range? 1. 0.5 ng/mL (0.63 nmol/L) 2. 0.8 ng/mL (1.02 nmol/L) 3. 0.9 ng/mL (1.14 nmol/L) 4. 2.2 ng/mL (2.8 nmol/L)

ANS: 4 The optimal therapeutic range for digoxin is 0.5 to 2.0 ng/mL. If the client is experiencing symptoms such as anorexia and is experiencing hypokalemia as evidenced by a low potassium level, digoxin toxicity is a concern. Therefore, option 4 is correct because it is outside of the therapeutic level and elevated.

The nurse is assisting to defibrillate a client in ventricular fibrillation. After placing the pads on the client's chest and before discharging the device, which intervention is a priority? 1. Ensure that the client has been intubated. 2. Set the defibrillator to the "synchronize" mode. 3. Administer an amiodarone bolus intravenously. 4. Confirm that the rhythm is ventricular fibrillation.

ANS: 4 Until the defibrillator is attached and charged, the client is resuscitated by using cardiopulmonary resuscitation. Once the defibrillator has been attached, the electrocardiogram is checked to verify that the rhythm is ventricular fibrillation or pulseless ventricular tachycardia. Leads also are checked for any loose connections. A nitroglycerin patch, if present, is removed. The client does not have to be intubated to be defibrillated. The machine is not set to the synchronous mode because there is no underlying rhythm with which to synchronize.

The nurse is caring for a client who had a resection of an abdominal aortic aneurysm yesterday. The client has an intravenous (IV) infusion at a rate of 150 mL/hr, unchanged for the last 10 hours. The client's urine output for the last 3 hours has been 90, 50, and 28 mL (28 mL is most recent). The client's BUN is 35 mg/dL (12.6 mmol/L), and the creatinine is 1.8 mg/dL. Which nursing action is the priority? 1. Check the serum albumin level. 2. Check the urine specific gravity. 3. Continue monitoring urine output. 4. Call the primary health care provider (PHCP).

ANS: 4 Urine output lower than 30 mL/hr is reported to the PHCP for urgent treatment. Following abdominal aortic aneurysm resection or repair, the nurse monitors the client for signs of acute kidney injury. Acute kidney injury can occur d/t blood is lost during the surgery and, depending on the aneurysm location, the renal arteries may be hypoperfused for a short period during surgery.

A client with variant angina is scheduled to receive an oral CCB twice daily. Which statement by the client indicates the need for further teaching? 1. "I should notify my cardiologist if my feet or legs start to swell." 2. "I am supposed to report to my cardiologist if my pulse rate decreases below 60." 3. "Avoiding grapefruit juice will definitely be a challenge for me, since I usually drink it every morning with breakfast." 4. "My spouse told me that since I have developed this problem, we are going to stop walking in the mall every morning."

ANS: 4 Variant angina, or Prinzmetal's angina, is prolonged and severe and occurs at the same time each day, most often at rest. The pain is a result of coronary artery spasm. The treatment of choice is usually a calcium channel blocker, which relaxes and dilates the vascular smooth muscle, thus relieving the coronary artery spasm in variant angina. Adverse effects can include peripheral edema, hypotension, bradycardia, and heart failure. Grapefruit juice interacts with calcium channel blockers and should be avoided. If bradycardia occurs, the client should contact the primary health care provider or cardiologist. Clients should also be taught to change positions slowly to prevent orthostatic hypotension. Physical exertion does not cause this type of angina; therefore, the client should be able to continue morning walks with her or his spouse.

A client in sinus bradycardia, with a heart rate of 45 beats per minute and blood pressure of 82/60 mm Hg, reports dizziness. Which intervention should the nurse anticipate will be prescribed? 1. Administer digoxin. 2. Defibrillate the client. 3. Continue to monitor the client. 4. Prepare for transcutaneous pacing.

ANS: 4 Sinus bradycardia is noted with a heart rate less than 60 beats per minute. This rhythm becomes a concern when the client becomes symptomatic. Hypotension and dizziness are signs of decreased cardiac output. Transcutaneous pacing provides a temporary measure to increase the heart rate and thus perfusion in the symptomatic client. Defibrillation is used for treatment of pulseless ventricular tachycardia and ventricular fibrillation. Digoxin will further decrease the client's heart rate. Continuing to monitor the client delays necessary intervention.

A nurse responds to an IV pump alarm related to increased pressure. Which action should the nurse take first? a. Check for kinking of the catheter. b. Flush the catheter with a thrombolytic enzyme. c. Get a new infusion pump. d. Remove the IV catheter.

ANS: A Fluid flow through the infusion system requires that pressure on the external side be greater than pressure at the catheter tip. Fluid flow can be slowed for many reasons. A common reason, and one that is easy to correct, is a kinked catheter. If this is not the cause of the pressure alarm, the nurse may have to ascertain whether a clot has formed inside the catheter lumen, or if the pump is no longer functional. Removal of the IV catheter and placement of a new IV catheter should be completed when no other option has resolved the problem.

A nurse teaches a client who is being discharged home with a peripherally inserted central catheter (PICC). Which statement should the nurse include in this clients teaching? a. Avoid carrying your grandchild with the arm that has the central catheter. b. Be sure to place the arm with the central catheter in a sling during the day. c. Flush the peripherally inserted central catheter line with normal saline daily. d. You can use the arm with the central catheter for most activities of daily living.

ANS: A A properly placed PICC (in the antecubital fossa or the basilic vein) allows the client considerable freedom of movement. Clients can participate in most activities of daily living; however, heavy lifting can dislodge the catheter or occlude the lumen. Although it is important to keep the insertion site and tubing dry, the client can shower. The device is flushed with heparin.

A nurse assesses clients on a cardiac unit. Which client should the nurse identify as being at greatest risk for the development of left-sided heart failure? a. A 36-year-old woman with aortic stenosis b. A 42-year-old man with pulmonary hypertension c. A 59-year-old woman who smokes cigarettes daily d. A 70-year-old man who had a cerebral vascular accident

ANS: A Although most people with heart failure will have failure that progresses from left to right, it is possible to have left-sided failure alone for a short period. It is also possible to have heart failure that progresses from right to left. Causes of left ventricular failure include mitral or aortic valve disease, coronary artery disease, and hypertension. Pulmonary hypertension and chronic cigarette smoking are risk factors for right ventricular failure. A cerebral vascular accident does not increase the risk of heart failure.

6. A nurse is teaching a client with heart failure who has been prescribed enalapril (Vasotec). Which statement should the nurse include in this clients teaching? a. Avoid using salt substitutes. b. Take your medication with food. c. Avoid using aspirin-containing products. d. Check your pulse daily.

ANS: A Angiotensin-converting enzyme (ACE) inhibitors such as enalapril inhibit the excretion of potassium. Hyperkalemia (fatal), should be taught to limit potassium intake & salt substitutes. ACE inhibitors do not need to be taken with food and have no impact on the clients pulse rate. Aspirin is often prescribed in conjunction with ACE inhibitors

A nurse admits a client who is experiencing an exacerbation of heart failure. Which action should the nurse take first? a. Assess the clients respiratory status. b. Draw blood to assess the clients serum electrolytes. c. Administer intravenous furosemide (Lasix). d. Ask the client about current medications.

ANS: A Assessment of respiratory and oxygenation status is the priority nursing intervention for the prevention of complications. Monitoring electrolytes, administering diuretics, and asking about current medications are important but do not take priority over assessing respiratory status

A nurse assesses a client with tachycardia. Which clinical manifestation requires immediate intervention by the nurse? a. Mid-sternal chest pain b. Increased urine output c. Mild orthostatic hypotension d. P wave touching the T wave

ANS: A Chest pain, possibly angina, indicates that tachycardia may be increasing the clients myocardial workload and oxygen demand to such an extent that normal oxygen delivery cannot keep pace. This results in myocardial hypoxia and pain. Increased urinary output and mild orthostatic hypotension are not life-threatening conditions and therefore do not require immediate intervention. The P wave touching the T wave indicates significant tachycardia and should be assessed to determine the underlying rhythm and cause; this is an important assessment but is not as critical as chest pain, which indicates cardiac cell death.

A client had an inferior wall myocardial infarction (MI). The nurse notes the clients ECG strip showing sinus bradycardia. What action by the nurse is most important? a. Assess the clients blood pressure and level of consciousness. b. Call the health care provider or the Rapid Response Team. c. Obtain a permit for an emergency temporary pacemaker insertion. d. Prepare to administer antidysrhythmic medication.

ANS: A Clients with an inferior wall MI often have bradycardia and blocks that lead to decreased perfusion. The nurse should first assess the clients hemodynamic status, including vital signs and level of consciousness. The client may or may not need the Rapid Response Team, a temporary pacemaker, or medication; there is no indication of this in the question.

A nurse cares for a client with right-sided heart failure. The client asks, Why do I need to weigh myself every day? How should the nurse respond? a. Weight is the best indication that you are gaining or losing fluid. b. Daily weights will help us make sure that youre eating properly. c. The hospital requires that all inpatients be weighed daily. d. You need to lose weight to decrease the incidence of heart failure.

ANS: A Daily weights are needed to document fluid retention or fluid loss. One liter of fluid equals 2.2 pounds. The other responses do not address the importance of monitoring fluid retention or loss.

A client is on a dopamine infusion via a peripheral line. What action by the nurse takes priority for safety? a. Assess the IV site hourly. b. Monitor the pedal pulses. c. Monitor the clients vital signs. d. Obtain consent for a central line.

ANS: A Dopamine should be infused through a central line to prevent extravasation and necrosis of tissue. If it needs to be run peripherally, the nurse assesses the site hourly for problems. When the client is getting the central line, ensuring informed consent is on the chart is a priority. But at this point, the client has only a peripheral line, so caution must be taken to preserve the integrity of the clients integumentary system. Monitoring pedal pulses and vital signs give indications as to how well the drug is working.

An emergency room nurse obtains the health history of a client. Which statement by the client should alert the nurse to the occurrence of heart failure? a. I get short of breath when I climb stairs. b. I see halos floating around my head. c. I have trouble remembering things. d. I have lost weight over the past month.

ANS: A Dyspnea on exertion is an early manifestation of heart failure and is associated with an activity such as stair climbing. The other findings are not specific to early occurrence of heart failure.

A client had a femoropopliteal bypass graft with a synthetic graft. What action by the nurse is most important to prevent wound infection? a. Appropriate hand hygiene before giving care b. Assessing the clients temperature every 4 hours c. Clean technique when changing dressings d. Monitoring the clients daily white blood cell count

ANS: A Hand hygiene is the best way to prevent infections in hospitalized clients. Dressing changes should be done with sterile technique. Assessing vital signs and white blood cell count will not prevent infection.

A nurse assesses a client who is recovering from a myocardial infarction. The clients pulmonary artery pressure reading is 25/12 mm Hg. Which action should the nurse take first? a. Compare the results with previous pulmonary artery pressure readings. b. Increase the intravenous fluid rate because these readings are low. c. Immediately notify the health care provider of the elevated pressures. d. Document the finding in the clients chart as the only action.

ANS: A Normal pulmonary artery pressures range from 15 to 26 mm Hg for systolic and from 5 to 15 mm Hg for diastolic. Although this clients readings are within normal limits, the nurse needs to assess any trends that may indicate a need for medical treatment to prevent complications. There is no need to increase intravenous fluids or notify the provider.

An older adult is on cardiac monitoring after a myocardial infarction. The client shows frequent dysrhythmias. What action by the nurse is most appropriate? a. Assess for any hemodynamic effects of the rhythm. b. Prepare to administer antidysrhythmic medication. c. Notify the provider or call the Rapid Response Team. d. Turn the alarms off on the cardiac monitor.

ANS: A Older clients may have dysrhythmias due to age-related changes in the cardiac conduction system. They may have no significant hemodynamic effects from these changes. The nurse should first assess for the effects of the dysrhythmia before proceeding further. The alarms on a cardiac monitor should never be shut off. The other two actions may or may not be needed.

A nurse teaches a client who experiences occasional premature atrial contractions (PACs) accompanied by palpitations that resolve spontaneously without treatment. Which statement should the nurse include in this clients teaching? a. Minimize or abstain from caffeine. b. Lie on your side until the attack subsides. c. Use your oxygen when you experience PACs. d. Take amiodarone (Cordarone) daily to prevent PACs.

ANS: A PACs usually have no hemodynamic consequences. For a client experiencing infrequent PACs, the nurse should explore possible lifestyle causes, such as excessive caffeine intake and stress. Lying on the side will not prevent or resolve PACs. Oxygen is not necessary. Although medications may be needed to control symptomatic dysrhythmias, for infrequent PACs, the client first should try lifestyle changes to control them.

While assessing a clients peripheral IV site, the nurse observes a streak of red along the vein path and palpates a 4-cm venous cord. How should the nurse document this finding? a. Grade 3 phlebitis at IV site b. Infection at IV site c. Thrombosed area at IV site d. Infiltration at IV site

ANS: A The presence of a red streak and palpable cord indicates grade 3 phlebitis. No information in the description indicates that infection, thrombosis, or infiltration is present.

While assessing a client on a cardiac unit, a nurse identifies the presence of an S3 gallop. Which action should the nurse take next? a. Assess for symptoms of left-sided heart failure. b. Document this as a normal finding. c. Call the health care provider immediately. d. Transfer the client to the intensive care unit.

ANS: A The presence of an S3 gallop is an early diastolic filling sound indicative of increasing left ventricular pressure and left ventricular failure. The other actions are not warranted.

A client with a history of heart failure and hypertension is in the clinic for a follow-up visit. The client is on lisinopril (Prinivil) and warfarin (Coumadin). The client reports new-onset cough. What action by the nurse is most appropriate? a. Assess the clients lung sounds and oxygenation. b. Instruct the client on another antihypertensive. c. Obtain a set of vital signs and document them. d. Remind the client that cough is a side effect of Prinivil.

ANS: A This client could be having an exacerbation of heart failure or be experiencing a side effect of lisinopril (and other angiotensin-converting enzyme inhibitors). The nurse should assess the clients lung sounds and other signs of oxygenation first. The client may or may not need to switch antihypertensive medications. Vital signs and documentation are important, but the nurse should assess the respiratory system first. If the cough turns out to be a side effect, reminding the client is appropriate, but then more action needs to be taken.

A nurse cares for a client who is on a cardiac monitor. The monitor displayed the rhythm shown below: Which action should the nurse take first? a. Assess airway, breathing, and level of consciousness. b. Administer an amiodarone bolus followed by a drip. c. Cardiovert the client with a biphasic defibrillator. d. Begin cardiopulmonary resuscitation (CPR).

ANS: A Ventricular tachycardia occurs with repetitive firing of an irritable ventricular ectopic focus, usually at a rate of 140 to 180 beats/min or more. Ventricular tachycardia is a lethal dysrhythmia. The nurse should first assess if the client is alert and breathing. Then the nurse should call a Code Blue and begin CPR. If this client is pulseless, the treatment of choice is defibrillation. Amiodarone is the antidysrhythmic of choice, but it is not the first action.

1A nurse assesses a client who had a myocardial infarction and is hypotensive. Which additional assessment finding should the nurse expect? a. Heart rate of 120 beats/min b. Cool, clammy skin c. Oxygen saturation of 90% d. Respiratory rate of 8 breaths/min

ANS: A When a client experiences hypotension, baroreceptors in the aortic arch sense a pressure decrease in the vessels. The parasympathetic system responds by lessening the inhibitory effect on the sinoatrial node. This results in an increase in heart rate and respiratory rate. This tachycardia is an early response and is seen even when blood pressure is not critically low. An increased heart rate and respiratory rate will compensate for the low blood pressure and maintain oxygen saturations and perfusion. The client may not be able to compensate for long, and decreased oxygenation and cool, clammy skin will occur later.

A nurse assists with the cardioversion of a client experiencing acute atrial fibrillation. Which action should the nurse take prior to the initiation of cardioversion? a. Administer intravenous adenosine. b. Turn off oxygen therapy. c. Ensure a tongue blade is available. d. Position the client on the left side.

ANS: B For safety during cardioversion, the nurse should turn off any oxygen therapy to prevent fire. The other interventions are not appropriate for a cardioversion. The client should be placed in a supine position.

A registered nurse (RN) delegates client care to an experienced licensed practical nurse (LPN). Which standards should guide the RN when delegating aspects of IV therapy to the LPN? (Select all that apply.) a. State Nurse Practice Act b. The facilitys Policies and Procedures manual c. The LPNs level of education and experience d. The Joint Commissions goals and criterion e. Client needs and prescribed orders

ANS: A, B The state Nurse Practice Act will have the information the RN needs, and in some states, LPNs are able to perform specific aspects of IV therapy. However, in a client care situation, it may be difficult and time-consuming to find it and read what LPNs are permitted to do, so another good solution would be for the nurse to check facility policy and follow it.

A nurse assesses a client who is recovering from a heart transplant. Which assessment findings should alert the nurse to the possibility of heart transplant rejection? (Select all that apply.) a. Shortness of breath b. Abdominal bloating c. New-onset bradycardia d. Increased ejection fraction e. Hypertension

ANS: A, B, C Clinical manifestations of heart transplant rejection include SOB, fatigue, fluid gain, abd bloating, new-onset bradycardia, hypotension, a-fibrillation or flutter, activity intolerance, and decreased ejection fraction.

A nurse is caring for a client with a history of renal insufficiency who is scheduled for a cardiac catheterization. Which actions should the nurse take prior to the catheterization? (Select all that apply.) a. Assess for allergies to iodine. b. Administer intravenous fluids. c. Assess blood urea nitrogen (BUN) and creatinine results. d. Insert a Foley catheter. e. Administer a prophylactic antibiotic. f. Insert a central venous catheter.

ANS: A, B, C If the client has kidney disease (as indicated by BUN and creatinine results), fluids and Mucomyst may be given 12 to 24 hours before the procedure for renal protection. The client should be assessed for allergies to iodine, including shellfish; the contrast medium used during the catheterization contains iodine. A Foley catheter and central venous catheter are not required for the procedure and would only increase the clients risk for infection. Prophylactic antibiotics are not administered prior to a cardiac catheterization.

A nurse assesses clients on a cardiac unit. Which clients should the nurse identify as at greatest risk for the development of acute pericarditis? (Select all that apply.) a. A 36-year-old woman with systemic lupus erythematosus (SLE) b. A 42-year-old man recovering from coronary artery bypass graft surgery c. A 59-year-old woman recovering from a hysterectomy d. An 80-year-old man with a bacterial infection of the respiratory tract e. An 88-year-old woman with a stage III sacral ulcer

ANS: A, B, D Acute pericarditis is most commonly associated acute exacerbations of systemic connective tissue disease, including SLE; with Dresslers syndrome (sac inflammation after cardiac surgery or a MI); and with infective organisms, including bacterial, viral, and fungal infections. Abdominal and reproductive surgeries and pressure ulcers do not increase clients risk for acute pericarditis.

A nurse is caring for a client on IV infusion of heparin. What actions does this nurse include in the clients plan of care? (Select all that apply.) a. Assess the client for bleeding. b. Monitor the daily activated partial thromboplastin time (aPTT) results. c. Stop the IV for aPTT above baseline. d. Use an IV pump for the infusion. e. Weigh the client daily on the same scale.

ANS: A, B, D Assessing for bleeding, monitoring aPTT, and using an IV pump for the infusion are all important safety measures for heparin to prevent injury from bleeding. The aPTT needs to be 1.5 to 2 times normal in order to demonstrate that the heparin is therapeutic. Weighing the client is not related.

A nurse prepares to administer a blood transfusion to a client, and checks the blood label with a second registered nurse using the International Society of Blood Transfusion (ISBT) universal bar-coding system to ensure the right blood for the right client. Which components must be present on the blood label in bar code and in eye-readable format? (Select all that apply.) a. Unique facility identifier b. Lot number related to the donor c. Name of the client receiving blood d. ABO group and Rh type of the donor e. Blood type of the client receiving blood

ANS: A, B, D The ISBT universal bar-coding system includes four components: (1) the unique facility identifier, (2) the lot number relating to the donor, (3) the product code, and (4) the ABO group and Rh type of the donor.

A nurse assists with the insertion of a central vascular access device. Which actions should the nurse ensure are completed to prevent a catheter-related bloodstream infection? (Select all that apply.) a. Include a review for the need of the device each day in the clients plan of care. b. Remind the provider to perform hand hygiene prior to starting the procedure. c. Cleanse the preferred site with alcohol and let it dry completely before insertion. d. Ask everyone in the room to wear a surgical mask during the procedure. e. Plan to complete a sterile dressing change on the device every day.

ANS: A, B, D The central vascular access device bundle to prevent catheter-related bloodstream infections includes using a checklist during insertion, performing hand hygiene before inserting the catheter and anytime someone touches the catheter, using chlorhexidine to disinfect the skin at the site of insertion, using preferred sites, and reviewing the need for the catheter every day. The practitioner who inserts the device should wear sterile gloves, gown and mask, and anyone in the room should wear a mask. A sterile dressing change should be completed per organizational policy, usually every 7 days and as needed.

A nurse prepares to discharge a client who has heart failure. Which questions to ensure this clients safety prior to discharge? (SAA) a. Are your bedroom and bathroom on the first floor? b. What social support do you have at home? c. Will you be able to afford your oxygen therapy? d. What spiritual beliefs may impact your recovery? e. Are you able to accurately weigh yourself at home?

ANS: A, B, D To ensure safety upon discharge, the nurse should assess for structural barriers to functional ability, such as stairs. The nurse should also assess the clients available social support, which may include family, friends, and home health services. The clients ability to adhere to medication and treatments, including daily weights, should also be reviewed. The other questions do not address the clients safety upon discharge.

pain of a (MI) differs from stable angina in what ways? (Select all that apply.) a. Accompanied by shortness of breath b. Feelings of fear or anxiety c. Lasts less than 15 minutes d. No relief from taking nitroglycerin e. Pain occurs without known cause

ANS: A, B, D, E The pain from an MI is often accompanied by shortness of breath and fear or anxiety. It lasts longer than 15 minutes and is not relieved by nitroglycerin. It occurs without a known cause such as exertion.

A nurse teaches a client with a new permanent pacemaker. Which instructions should the nurse include in this clients teaching? (Select all that apply.) a. Until your incision is healed, do not submerge your pacemaker. Only take showers. b. Report any pulse rates lower than pacemaker settings. c. If you feel weak, apply pressure over your generator. d. Have your pacemaker turned off before having magnetic resonance imaging (MRI). e. Do not lift your left arm above the level of your shoulder for 8 weeks.

ANS: A, B, E The client should not submerge in water until the site has healed; after the incision is healed, the client may take showers or baths without concern for the pacemaker. The client should be instructed to report changes in heart rate or rhythm, such as rates lower than the pacemaker setting or greater than 100 beats/min. The client should be advised of restrictions on physical activity for 8 weeks to allow the pacemaker to settle in place. The client should never apply pressure over the generator and should avoid tight clothing. The client should never have MRI because, whether turned on or off, the pacemaker contains metal. The client should be advised to inform all health care providers that he or she has a pacemaker.

A nurse evaluates laboratory results for a client with heart failure. Which results should the nurse expect? (Select all that apply.) a. Hematocrit: 32.8% b. Serum sodium: 130 mEq/L c. Serum potassium: 4.0 mEq/L d. Serum creatinine: 1.0 mg/dL e. Proteinuria f. Microalbuminuria

ANS: A, B, E, F A hematocrit of 32.8% is low (34% - 52%), indicating a dilutional ratio of red blood cells to fluid. A serum sodium of 130 mEq/L is low d/t hemodilution. Microalbuminuria and proteinuria are present, indicating a decrease in renal filtration. These are early warning signs of decreased compliance of the heart. The potassium level is on the high side of normal and the serum creatinine level is normal.

A nurse assesses a client who has a peripherally inserted central catheter (PICC). For which complications should the nurse assess? (Select all that apply.) a. Phlebitis b. Pneumothorax c. Thrombophlebitis d. Excessive bleeding e. Extravasation

ANS: A, C Although the complication rate with PICCs is fairly low, the most common complications are phlebitis, thrombophlebitis, and catheter-related bloodstream infection. Pneumothorax, excessive bleeding, and extravasation are not common complications.

A client is being discharged on warfarin (Coumadin) therapy. What discharge instructions is the nurse required to provide? (Select all that apply.) a. Dietary restrictions b. Driving restrictions c. Follow-up laboratory monitoring d. Possible drug-drug interactions e. Reason to take medication

ANS: A, C, D, E The Joint Commissions Core Measures state that clients being discharged on warfarin need instruction on follow-up monitoring, dietary restrictions, drug-drug interactions, and reason for compliance. Driving is typically not restricted.

A nurse reviews a clients laboratory results. Which findings should alert the nurse to the possibility of atherosclerosis? (Select all that apply.) a. Total cholesterol: 280 mg/dL b. High-density lipoprotein cholesterol: 50 mg/dL c. Triglycerides: 200 mg/dL d. Serum albumin: 4 g/dL e. Low-density lipoprotein cholesterol: 160 mg/dL

ANS: A, C, E A lipid panel is often used to screen for cardiovascular risk. Total cholesterol (< 200 mg/dL), triglycerides (m: 40-160; f: 35-135 mg/dL), and LDL ( < 130 mg/dL) levels are all high, indicating higher risk for cardiovascular disease. HDL (m: > 45; f: > 55) is within the normal range for both males and females. Serum albumin (3.5-5.0 g/dL) is not assessed for atherosclerosis.

A nurse cares for a client who is recovering from a right-sided heart catheterization. For which complications of this procedure should the nurse assess? (Select all that apply.) a. Thrombophlebitis b. Stroke c. Pulmonary embolism d. Myocardial infarction e. Cardiac tamponade

ANS: A, C, E Complications from a right-sided heart catheterization include thrombophlebitis, pulmonary embolism, vagal response, and Cardiac tamponade.

A nurse cares for a client with congestive heart failure who has a regular cardiac rhythm of 128 beats/min. For which physiologic alterations should the nurse assess? (Select all that apply.) a. Decrease in cardiac output b. Increase in cardiac output c. Decrease in blood pressure d. Increase in blood pressure e. Decrease in urine output f. Increase in urine output

ANS: A, D, E Elevated heart rates in a healthy client initially cause blood pressure and cardiac output to increase. However, in a client who has congestive heart failure or a client with long-term tachycardia, ventricular filling time, cardiac output, and blood pressure eventually decrease. As cardiac output and blood pressure decrease, urine output will fall.

After teaching a client with congestive heart failure (CHF), the nurse assesses the clients understanding. Which client statements indicate a correct understanding of the teaching related to nutritional intake? (Select all that apply.) a. Ill read the nutritional labels on food items for salt content. b. I will drink at least 3 liters of water each day. c. Using salt in moderation will reduce the workload of my heart. d. I will eat oatmeal for breakfast instead of ham and eggs. e. Substituting fresh vegetables for canned ones will lower my salt intake.

ANS: A, D, E Nutritional therapy for a client with CHF is focused on decreasing sodium and water retention to decrease the workload of the heart. The client should be taught to read nutritional labels on all food items, omit table salt and foods high in sodium (e.g., ham and canned foods), & limit water intake to 2 L/day.

A nurse assesses a client admitted to cardiac unit. Which statement alerts the RN to right-sided HF? a. I sleep w/ 4 pillows at night b. Shoes fit tight lately c. Wake up coughing every night d. Trouble catching breath

ANS: B Signs of systemic congestion occur w/ RHF: fluid retention, peripheral edema, Nocturia.

A nurse cares for a client recovering from prosthetic valve replacement surgery. The client asks, Why will I need to take anticoagulants for the rest of my life? How should the nurse respond? a. The prosthetic valve places you at greater risk for a heart attack. b. Blood clots form more easily in artificial replacement valves. c. The vein taken from your leg reduces circulation in the leg. d. The surgery left a lot of small clots in your heart and lungs.

ANS: B Synthetic valve prostheses and scar tissue provide surfaces on which platelets can aggregate easily and initiate the formation of blood clots.

The nurse is caring for a client on the medical-surgical unit who suddenly becomes unresponsive and has no pulse. The cardiac monitor shows ventricular fibrillation. After calling for assistance and a defibrillator, which action should the nurse take next? a. Perform a pericardial thump. b. Initiate cardiopulmonary resuscitation (CPR). c. Start an 18-gauge intravenous line. d. Ask the clients family about code status.

ANS: B This is a lethal rhythm that is best treated with immediate defibrillation. While the nurse is waiting for the defibrillator to arrive, the nurse should start CPR. A pericardial thump is not a treatment for ventricular fibrillation. If the client does not already have an IV, other members of the team can insert one after defibrillation. The clients code status should already be known by the nurse prior to this event.

A nurse is caring for a client who has just had a central venous access line inserted. Which action should the nurse take next? Begin the prescribed infusion via the new access. b. Ensure an x-ray is completed to confirm placement. c. Check medication calculations with a second RN. d. Make sure the solution is appropriate for a central line.

ANS: B A central venous access device, once placed, needs an x-ray confirmation of proper placement before it is used. The bedside nurse would be responsible for beginning the infusion once placement has been verified. Any IV solution can be given through a central line.

After administering newly prescribed captopril (Capoten) to a client with heart failure, the nurse implements interventions to decrease complications. Which priority intervention should the nurse implement for this client? a. Provide food to decrease nausea and aid in absorption. b. Instruct the client to ask for assistance when rising from bed. c. Collaborate with unlicensed assistive personnel to bathe the client. d. Monitor potassium levels and check for symptoms of hypokalemia.

ANS: B Administration of the first dose of angiotensin-converting enzyme (ACE) inhibitors is often associated with hypotension, usually termed first- dose effect. The nurse should instruct the client to seek assistance before arising from bed to prevent injury from postural hypotension. ACE inhibitors do not need to be taken with food. Collaboration with unlicensed assistive personnel to provide hygiene is not a priority. The client should be encouraged to complete activities of daily living as independently as possible. The nurse should monitor for hyperkalemia, not hypokalemia, especially if the client has renal insufficiency secondary to heart failure.

A client received tissue plasminogen activator (t-PA) after a myocardial infarction and now is on an intravenous infusion of heparin. The clients spouse asks why the client needs this medication. What response by the nurse is best? a. The t-PA didnt dissolve the entire coronary clot. b. The heparin keeps that artery from getting blocked again. c. Heparin keeps the blood as thin as possible for a longer time. d. The heparin prevents a stroke from occurring as the t-PA wears off.

ANS: B After the original intracoronary clot has dissolved, large amounts of thrombin are released into the bloodstream, increasing the chance of the vessel reoccluding. The other statements are not accurate. Heparin is not a blood thinner, although laypeople may refer to it as such.

A nurse is caring for a client who is having a subclavian central venous catheter inserted. The client begins to report chest pain and difficulty breathing. After administering oxygen, which action should the nurse take next? a. Administer a sublingual nitroglycerin tablet. b. Prepare to assist with chest tube insertion. c. Place a sterile dressing over the IV site. d. Re-position the client into the Trendelenburg position.

ANS: B An insertion-related complication of central venous catheters is a pneumothorax. Signs and symptoms of a pneumothorax include chest pain and dyspnea. Treatment includes removing the catheter, administering oxygen, and placing a chest tube. Pain is caused by the pneumothorax, which must be taken care of with a chest tube insertion. Use of a sterile dressing and placement of the client in a Trendelenburg position are not indicated for the primary problem of a pneumothorax.

A client had a percutaneous transluminal coronary angioplasty for peripheral arterial disease. What assessment finding by the nurse indicates a priority outcome for this client has been met? a. Pain rated as 2/10 after medication b. Distal pulse on affected extremity 2+/4+ c. Remains on bedrest as directed d. Verbalizes understanding of procedure

ANS: B Assessing circulation distal to the puncture site is a critical nursing action. A pulse of 2+/4+ indicates good perfusion. Pain control, remaining on bedrest as directed after the procedure, and understanding are all important, but do not take priority over perfusion.

A nurse is caring for a client with a peripheral vascular access device who is experiencing pain, redness, and swelling at the site. After removing the device, which action should the nurse take to relieve pain? a. Administer topical lidocaine to the site. b. Place warm compresses on the site. c. Administer prescribed oral pain medication. d. Massage the site with scented oils.

ANS: B At the first sign of phlebitis, the catheter should be removed and warm compresses used to relieve pain. The other options are not appropriate for this type of pain.

A nurse assesses a client who has mitral valve regurgitation. For which cardiac dysrhythmia should the nurse assess? a. Preventricular contractions b. Atrial fibrillation c. Symptomatic bradycardia d. Sinus tachycardia

ANS: B Atrial fibrillation is a clinical manifestation of mitral valve regurgitation and stenosis. Preventricular contractions and bradycardia are not associated with valvular problems. These are usually identified in clients with electrolyte imbalances, myocardial infarction, and sinus node problems. Sinus tachycardia is a manifestation of aortic regurgitation due to a decrease in cardiac output.

A nurse is assessing clients on a medical-surgical unit. Which client should the nurse identify as being at greatest risk for atrial fibrillation? a. A 45-year-old who takes an aspirin daily b. A 50-year-old who is post coronary artery bypass graft surgery c. A 78-year-old who had a carotid endarterectomy d. An 80-year-old with chronic obstructive pulmonary disease .

ANS: B Atrial fibrillation occurs commonly in clients with cardiac disease and is a common occurrence after CABG surgery. The other conditions do not place these clients at higher risk for atrial fibrillation

A nurse evaluates prescriptions for a client with chronic atrial fibrillation. Which medication should the nurse expect to find on this clients medication administration record to prevent a common complication of this condition? a. Sotalol (Betapace) b. Warfarin (Coumadin) c. Atropine (Sal-Tropine) d. Lidocaine (Xylocaine)

ANS: B Atrial fibrillation puts clients at risk for developing emboli. Clients at risk for emboli are treated with anticoagulants, such as heparin, enoxaparin, or warfarin. Sotalol, atropine, and lidocaine are not appropriate for this complication.

A nurse cares for a client who has a heart rate averaging 56 beats/min with no adverse symptoms. Which activity modification should the nurse suggest to avoid further slowing of the heart rate? a. Make certain that your bath water is warm. b. Avoid straining while having a bowel movement. c. Limit your intake of caffeinated drinks to one a day. d. Avoid strenuous exercise such as running.

ANS: B Bearing down strenuously during a bowel movement is one type of Valsalva maneuver, which stimulates the vagus nerve and results in slowing of the heart rate. Such a response is not desirable in a person who has bradycardia.

A client is receiving an infusion of alteplase (Activase) for an intra-arterial clot. The client begins to mumble and is disoriented. What action by the nurse takes priority? a. Assess the clients neurologic status. b. Notify the Rapid Response Team. c. Prepare to administer vitamin K. d. Turn down the infusion rate.

ANS: B Clients on fibrinolytic therapy are at high risk of bleeding. The sudden onset of neurologic signs may indicate the client is having a hemorrhagic stroke. The nurse does need to complete a thorough neurological examination, but should first call the Rapid Response Team based on the clients manifestations. The nurse notifies the Rapid Response Team first. Vitamin K is not the antidote for this drug. Turning down the infusion rate will not be helpful if the client is still receiving any of the drug.

After teaching a client who is being discharged home after mitral valve replacement surgery, the nurse assesses the clients understanding. Which client statement indicates a need for additional teaching? a. Ill be able to carry heavy loads after 6 months of rest. b. I will have my teeth cleaned by my dentist in 2 weeks. c. I must avoid eating foods high in vitamin K, like spinach. d. I must use an electric razor instead of a straight razor to shave.

ANS: B Clients who have defective or repaired valves are at high risk for endocarditis. The client who has had valve surgery should avoid dental procedures for 6 months d/t risk for endocarditis. When undergoing a mitral valve replacement surgery, pt is placed on anticoagulant therapy to prevent vegetation forming on the new valve. Clients on anticoagulant therapy should be instructed on bleeding precautions, including using an electric razor. If the client is prescribed warfarin, the client should avoid foods high in vitamin K. Clients recovering from open heart valve replacements should not carry anything heavy for 6 months while the chest incision and muscle heal

A nurse assesses a client with atrial fibrillation. Which manifestation should alert the nurse to the possibility of a serious complication from this condition? a. Sinus tachycardia b. Speech alterations c. Fatigue d. Dyspnea with activity

ANS: B Clients with atrial fibrillation are at risk for embolic stroke. Evidence of embolic events includes changes in mentation, speech, sensory function, and motor function. Clients with atrial fibrillation often have a rapid ventricular response as a result. Fatigue is a nonspecific complaint. Clients with atrial fibrillation often have dyspnea as a result of the decreased cardiac output caused by the rhythm disturbance.

A nurse is caring for a client who is receiving an epidural infusion for pain management. Which assessment finding requires immediate intervention from the nurse? a. Redness at the catheter insertion site b. Report of headache and stiff neck c. Temperature of 100.1 F (37.8 C) d. Pain rating of 8 on a scale of 0 to 10

ANS: B Complications of epidural therapy include infection, bleeding, leakage of cerebrospinal fluid, occlusion of the catheter lumen, and catheter migration. Headache, neck stiffness, and a temperature higher than 101 F are signs of meningitis and should be reported to the provider immediately. The other findings are important but do not require immediate intervention.

A nurse is in charge of the coronary intensive care unit. Which client should the nurse see first? a. Client on a nitroglycerin infusion at 5 mcg/min, not titrated in the last 4 hours b. Client who is 1 day post coronary artery bypass graft, blood pressure 180/100 mm Hg c. Client who is 1 day post percutaneous coronary intervention, going home this morning d. Client who is 2 days post coronary artery bypass graft, became dizzy this a.m. while walking

ANS: B Hypertension after coronary artery bypass graft surgery can be dangerous because it puts too much pressure on the suture lines and can cause bleeding. The charge nurse should see this client first. The client who became dizzy earlier should be seen next. The client on the nitroglycerin drip is stable. The client going home can wait until the other clients are cared for.

A client has hemodynamic monitoring after a myocardial infarction. What safety precaution does the nurse implement for this client? a. Document pulmonary artery wedge pressure (PAWP) readings and assess their trends. b. Ensure the balloon does not remain wedged. c. Keep the client on strict NPO status. d. Maintain the client in a semi-Fowlers position.

ANS: B If the balloon remains inflated, it can cause pulmonary infarction or rupture. The nurse should ensure the balloon remains deflated between PAWP readings. Documenting PAWP readings and assessing trends is an important nursing action related to hemodynamic monitoring, but is not specifically related to safety. The client does not have to be NPO while undergoing hemodynamic monitoring. Positioning may or may not affect readings.

The nurse is caring for a client with a chest tube after a coronary artery bypass graft. The drainage slows significantly. What action by the nurse is most important? a. Increase the setting on the suction. b. Notify the provider immediately. c. Re-position the chest tube. d. Take the tubing apart to assess for clots.

ANS: B If the drainage in the chest tube decreases significantly and dramatically, the tube may be blocked by a clot. This could lead to cardiac tamponade. The nurse should notify the provider immediately. The nurse should not independently increase the suction, re-position the chest tube, or take the tubing apart.

A nurse cares for a client with an intravenous temporary pacemaker for bradycardia. The nurse observes the presence of a pacing spike but no QRS complex on the clients electrocardiogram. Which action should the nurse take next? a. Administer intravenous diltiazem (Cardizem). b. Assess vital signs and level of consciousness. c. Administer sublingual nitroglycerin. d. Assess capillary refill and temperature.

ANS: B In temporary pacing, the wires are threaded onto the epicardial surface of the heart and exit through the chest wall. The pacemaker spike should be followed immediately by a QRS complex. Pacing spikes seen without subsequent QRS complexes imply loss of capture. If there is no capture, then there is no ventricular depolarization and contraction. The nurse should assess for cardiac output via vital signs and level of consciousness. The other interventions would not determine if the client is tolerating the loss of capture.

A client with a history of type 2 diabetes is admitted to the hospital with chest pain. The client is scheduled for a cardiac cath. Which medication would need to be withheld for 24 hours before the procedure and for 48 hours after the procedure? a. Glipizide b. Metformin c. Repaglinide d. Regular insulin

ANS: B Metformin needs to be withheld 24 hours before and for 48 hours after cardiac catheterization because of the injection of contrast medium during the procedure. If the contrast medium affects kidney function, with metformin in the system the client would be at increased risk for lactic acidosis. The medications in the remaining options do not need to be withheld before and after cardiac catheterization.

A client with coronary artery disease (CAD) asks the nurse about taking fish oil supplements. What response by the nurse is best? a. Fish oil is contraindicated with most drugs for CAD. b. The best source is fish, but pills have benefits too. c. There is no evidence to support fish oil use with CAD. d. You can reverse CAD totally with diet and supplements.

ANS: B Omega-3 fatty acids have shown benefit in reducing lipid levels, in reducing the incidence of sudden cardiac death, and for stabilizing atherosclerotic plaque. The best source is fish three times a week or some fish oil supplements.

A client has presented to the emergency department with an acute myocardial infarction (MI). What action by the nurse is best to meet The Joint Commissions Core Measures outcomes? a. Obtain an electrocardiogram (ECG) now and in the morning. b. Give the client an aspirin. c. Notify the Rapid Response Team. d. Prepare to administer thrombolytics.

ANS: B The Joint Commissions Core Measures set for acute MI require that aspirin is administered when a client with MI presents to the emergency department or when an MI occurs in the hospital. A rapid ECG is vital, but getting another one in the morning is not part of the Core Measures set. The Rapid Response Team is not needed if an emergency department provider is available. Thrombolytics may or may not be needed.

A nurse cares for a client who has an 80% blockage of the right coronary artery (RCA) and is scheduled for bypass surgery. Which intervention should the nurse be prepared to implement while this client waits for surgery? a. Administration of IV furosemide (Lasix) b. Initiation of an external pacemaker c. Assistance with endotracheal intubation d. Placement of central venous access

ANS: B The RCA supplies the right atrium, the right ventricle, the inferior portion of the left ventricle, and the atrioventricular (AV) node. It also supplies the sinoatrial node in 50% of people. If the client totally occludes the RCA, the AV node would not function and the client would go into heart block, so emergency pacing should be available for the client. Furosemide, intubation, and central venous access will not address the primary complication of RCA occlusion, which is AV node malfunction.

A nurse assesses a client with pericarditis. Which assessment finding should the nurse expect to find? a. Heart rate that speeds up and slows down b. Friction rub at the left lower sternal border c. Presence of a regular gallop rhythm d. Coarse crackles in bilateral lung bases

ANS: B The client with pericarditis may present with a pericardial friction rub at the left lower sternal border. This sound is the result of friction from inflamed pericardial layers when they rub together. The other assessments are not related.

A nurse is caring for four clients. Which client should the nurse assess first? a. Client with an acute myocardial infarction, pulse 102 beats/min b. Client who is 1 hour post angioplasty, has tongue swelling and anxiety c. Client who is post coronary artery bypass, chest tube drained 100 mL/hr d. Client who is post coronary artery bypass, potassium 4.2 mEq/L

ANS: B The post-angioplasty client with tongue swelling and anxiety is exhibiting manifestations of an allergic reaction that could progress to anaphylaxis. The nurse should assess this client first. The client with a heart rate of 102 beats/min may have increased oxygen demands but is just over the normal limit for heart rate. The two post coronary artery bypass clients are stable.

A nurse auscultated heart tones on an older adult client and found an atrial gallop S4. Which action should the nurse take? a. Administer a diuretic. b. Document the finding. c. Decrease the IV flow rate. d. Evaluate the clients medications.

ANS: B The sound heard is an atrial gallop S4. An atrial gallop may be heard in older clients because of a stiffened ventricle. The nurse should document the finding, but no other intervention is needed at this time.

A nurse obtains the health history of a client who is newly admitted to the medical unit. Which statement by the client should alert the nurse to the presence of edema? a. I wake up to go to the bathroom at night. b. My shoes fit tighter by the end of the day. c. I seem to be feeling more anxious lately. d. I drink at least eight glasses of water a day.

ANS: B Weight gain can result from fluid accumulation in the interstitial spaces. This is known as edema. The nurse should note whether the client feels that his or her shoes or rings are tight, and should observe, when present, an indentation around the leg where the socks end. The other answers do not describe edema.

After assessing a client who is receiving an amiodarone intravenous infusion for unstable ventricular tachycardia, the nurse documents the findings and compares these with the previous assessment findings: client is asymptomatic & no evidence AV block needing pacing. which action should the nurse take? a. Stop the infusion and flush the IV. b. Slow the amiodarone infusion rate. c. Administer IV normal saline. d. Ask the client to cough and deep breathe.

ANS: B IV administration of amiodarone may cause bradycardia and atrioventricular (AV) block. The correct action for the nurse to take at this time is to slow the infusion, because the client is asymptomatic and no evidence reveals AV block that might require pacing. Abruptly ceasing the medication could allow fatal dysrhythmias to occur. The administration of IV fluids and encouragement of coughing and deep breathing exercises are not indicated, and will not increase the clients heart rate.

A nurse assesses a client who is diagnosed with infective endocarditis. Which assessment findings should the nurse expect? (SAA) a. Weight gain b. Night sweats c. Cardiac murmur d. Abdominal bloating e. Oslers nodes

ANS: B, C, E Clinical manifestations of infective endocarditis include recurrent fever, night sweats, malaise/fatigue, anorexia and weight loss, cardiac murmur, and Oslers nodes. Abdominal bloating is a manifestation of heart transplantation

An emergency room nurse assesses a female client. Which assessment findings should alert the nurse to request a prescription for an electrocardiogram? (Select all that apply.) a. Hypertension b. Fatigue despite adequate rest c. Indigestion d. Abdominal pain e. Shortness of breath

ANS: B, C, E Women may not have chest pain with myocardial infarction, but may feel discomfort or indigestion. They often present with a triad of symptomsindigestion or feeling of abdominal fullness, feeling of chronic fatigue despite adequate rest, and feeling unable to catch their breath. Frequently, women are not diagnosed and therefore are not treated adequately. Hypertension and abdominal pain are not associated with acute coronary syndrome.

A nurse prepares a client for a pharmacologic stress echocardiogram. Which actions should the nurse take when preparing this client for the procedure? (Select all that apply.) a. Assist the provider to place a central venous access device. b. Prepare for continuous blood pressure and pulse monitoring. c. Administer the clients prescribed beta blocker. d. Give the client nothing by mouth 3 to 6 hours before the procedure. e. Explain to the client that dobutamine will simulate exercise for this examination.

ANS: B, D, E Clients receiving a pharmacologic stress echocardiogram will need peripheral venous access and continuous blood pressure and pulse monitoring. The client must be NPO 3 to 6 hours prior to the procedure. Education about dobutamine, which will be administered during the procedure, should be performed. Beta blockers are often held prior to the procedure.

A nurse cares for a client who is recovering from a left-sided heart catheterization. For which complications of this procedure should the nurse assess? (Select all that apply.) a. Thrombophlebitis b. Stroke c. Pulmonary embolism d. Myocardial infarction e. Cardiac tamponade

ANS: B, D, E Complications from a left-sided heart catheterization include Stroke, myocardial infarction, cardiac tamponade.

A nurse assesses a client who is recovering after a coronary catheterization. Which assessment findings in the first few hours after the procedure require immediate action by the nurse? (Select all that apply.) a. Blood pressure of 140/88 mm Hg b. Serum potassium of 2.9 mEq/L c. Warmth and redness at the site d. Expanding groin hematoma e. Rhythm changes on the cardiac monitor

ANS: B, D, E In the first few hours postprocedure, the nurse monitors for complications such as bleeding from the insertion site, hypotension, acute closure of the vessel, dye reaction, hypokalemia, and dysrhythmias. The clients blood pressure is slightly elevated but does not need immediate action. Warmth and redness at the site would indicate an infection, but this would not be present in the first few hours.

A client is 1 day postoperative after a coronary artery bypass graft. What nonpharmacologic comfort measures does the nurse include when caring for this client? (Select all that apply.) a. Administer pain medication before ambulating. b. Assist the client into a position of comfort in bed. c. Encourage high-protein diet selections. d. Provide complementary therapies such as music. e. Remind the client to splint the incision when coughing.

ANS: B, D, E Nonpharmacologic comfort measures can include positioning, complementary therapies, and splinting the chest incision. Medications are not nonpharmacologic. Food choices are not comfort measures.

A client with a known abdominal aortic aneurysm reports dizziness and severe abdominal pain. The nurse assesses the clients blood pressure at 82/40 mm Hg. What actions by the nurse are most important? (Select all that apply.) a. Administer pain medication. b. Assess distal pulses every 10 minutes. c. Have the client sign a surgical consent. d. Notify the Rapid Response Team. e. Take vital signs every 10 minutes.

ANS: B, D, E This client may have a ruptured/rupturing aneurysm. The nurse should notify the Rapid Response team and perform frequent client assessments. Giving pain medication will lower the clients blood pressure even further. The nurse cannot have the client sign a consent until the physician has explained the procedure.

A client presents to the emergency department with an acute myocardial infarction (MI) at 1500 (3:00 PM). The facility has 24-hour cath lab abilities. By what time should the client have a percutaneous coronary intervention performed? a. 1530 (3:30 PM) b. 1600 (4:00 PM) c. 1630 (4:30 PM) d. 1700 (5:00 PM)

ANS: C Measures set for MI includes percutaneous coronary intervention within 90 minutes of diagnosis of myocardial infarction.

A telemetry nurse assesses a client with third-degree heart block who has wide QRS complexes and a heart rate of 35 beats/min on the cardiac monitor. Which assessment should the nurse complete next? a. Pulmonary auscultation b. Pulse strength and amplitude c. Level of consciousness d. Mobility and gait stability

ANS: C A HR ≤ 40 bpm with widened QRS complexes could have hemodynamic consequences. The client is at risk for inadequate cerebral perfusion. The nurse should assess for level of consciousness, light-headedness, confusion, syncope, and seizure activity. Although the other assessments should be completed, the clients level of consciousness is the priority.

A client is receiving an infusion of tissue plasminogen activator (t-PA). The nurse assesses the client to be disoriented to person, place, and time. What action by the nurse is best? a. Assess the clients pupillary responses. b. Request a neurologic consultation. c. Stop the infusion and call the provider. d. Take and document a full set of vital signs.

ANS: C A change in neurologic status in a client receiving t-PA could indicate intracranial hemorrhage. The nurse should stop the infusion and notify the provider immediately. A full assessment, including pupillary responses and vital signs, occurs next. The nurse may or may not need to call a neurologist.

After teaching a client who is recovering from a heart transplant to change positions slowly, the client asks, Why is this important? How should the nurse respond? a. Rapid position changes can create shear and friction forces, and tear internal vascular sutures. b. Your new vascular connections are more sensitive to position changes, causing dizziness. c. Your new heart is not connected to the nervous system and is unable to respond to decreases in blood pressure caused by position changes. d. While your heart is recovering, blood flow is diverted away from the brain, increasing stroke risk

ANS: C Because the new heart is denervated, the baroreceptor and other mechanisms that compensate for blood pressure drops caused by position changes do not function. This allows orthostatic hypotension to persist in the postoperative period. The other options are false statements and do not correctly address the clients question.

13.A nurse teaches a client who is prescribed a central vascular access device. Which statement should the nurse include in this clients teaching? a. You will need to wear a sling on your arm while the device is in place. b. There is no risk of infection because sterile technique will be used during insertion. c. Ask all providers to vigorously clean the connections prior to accessing the device. d. You will not be able to take a bath with this vascular access device.

ANS: C Clients should be actively engaged in the prevention of catheter-related bloodstream infections and taught to remind all providers to perform hand hygiene and vigorously clean connections prior to accessing the device. The other statements are incorrect.

A nurse administers prescribed adenosine (Adenocard) to a client. Which response should the nurse assess for as the expected therapeutic response? a. Decreased intraocular pressure b. Increased heart rate c. Short period of asystole d. Hypertensive crisis

ANS: C Clients usually respond to adenosine with a short period of asystole, bradycardia, hypotension, dyspnea, and chest pain. Adenosine has no conclusive impact on IOP.

A nurse assesses a client in an outpatient clinic. Which statement alerts the nurse to the possibility of left-sided heart failure? a. I have been drinking more water than usual. b. I am awakened by the need to urinate at night. c. I must stop halfway up the stairs to catch my breath. d. I have experienced blurred vision on several occasions.

ANS: C Clients with LHF report weakness or fatigue while performing ADLs, dyspnea, Orthopnea, pulmonary crackles, confusion, restless and cough that worsens at night. This occurs as fluid moves into the alveoli. Thirst and blurred vision are not related to heart failure.

A nurse assesses an older adult client who is experiencing a myocardial infarction. Which clinical manifestation should the nurse expect? a. Excruciating pain on inspiration b. Left lateral chest wall pain c. Disorientation and confusion d. Numbness and tingling of the arm

ANS: C In older adults, disorientation or confusion may be the major manifestation of myocardial infarction caused by poor cardiac output. Pain manifestations and numbness and tingling of the arm could also be related to the myocardial infarction. However, the nurse should be more concerned about the new onset of disorientation or confusion caused by decreased perfusion.

A nurse assesses a client 2 hours after a cardiac angiography via the left femoral artery. The nurse notes that the left pedal pulse is weak. Which action should the nurse take? a. Elevate the leg and apply a sandbag to the entrance site. b. Increase the flow rate of intravenous fluids. c. Assess the color and temperature of the left leg. d. Document the finding as left pedal pulse of +1/4.

ANS: C Loss of a pulse distal to an angiography entry site is serious, indicating a possible arterial obstruction. The pulse may be faint because of edema. The left pulse should be compared with the right, and pulses should be compared with previous assessments, especially before the procedure. Assessing color (pale, cyanosis) and temperature (cool, cold) will identify a decrease in circulation. Once all peripheral and vascular assessment data are acquired, the primary health care provider should be notified. Simply documenting the findings is inappropriate. The leg should be positioned below the level of the heart or dangling to increase blood flow to the distal portion of the leg. Increasing intravenous fluids will not address the clients problem.

A nurse assesses an older adult client who has multiple chronic diseases. The clients heart rate is 48 beats/min. Which action should the nurse take first? a. Document the finding in the chart. b. Initiate external pacing. c. Assess the clients medications. d. Administer 1 mg of atropine.

ANS: C Pacemaker cells in the conduction system decrease in number as a person ages, resulting in bradycardia. The nurse should check the medication reconciliation for medications that might cause such a drop in heart rate, then should inform the health care provider. Documentation is important, but it is not the priority action. The heart rate is not low enough for atropine or an external pacemaker to be needed

A client had an acute myocardial infarction. What assessment finding indicates to the nurse that a significant complication has occurred? a. Blood pressure that is 20 mm Hg below baseline b. Oxygen saturation of 94% on room air c. Poor peripheral pulses and cool skin d. Urine output of 1.2 mL/kg/hr for 4 hours

ANS: C Poor peripheral pulses and cool skin may be signs of impending cardiogenic shock and should be reported immediately. A blood pressure drop of 20 mm Hg is not worrisome. An oxygen saturation of 94% is just slightly below normal. A urine output of 1.2 mL/kg/hr for 4 hours is normal.

A nurse assesses clients on a medical-surgical unit. Which client should the nurse identify as having the greatest risk for cardiovascular disease? a. An 86-year-old man with a history of asthma b. A 32-year-old Asian-American man with colorectal cancer c. A 45-year-old American Indian woman with diabetes mellitus d. A 53-year-old postmenopausal woman who is on hormone therapy

ANS: C The incidence of coronary artery disease and hypertension is higher in American Indians than in whites or Asian Americans. Diabetes mellitus increases the risk for hypertension and coronary artery disease in people of any race or ethnicity. Asthma, colorectal cancer, and hormone therapy do not increase risk for cardiovascular disease.

A nurse performs an admission assessment on a 75-year-old client with multiple chronic diseases. The clients blood pressure is 135/75 mm Hg and oxygen saturation is 94% on 2 liters per nasal cannula. The nurse assesses the clients rhythm on the cardiac monitor and observes bradycardia. Which action should the nurse take first? a. Begin external temporary pacing. b. Assess peripheral pulse strength. c. Ask the client what medications he or she takes. d. Administer 1 mg of atropine.

ANS: C This client is stable and therefore does not require any intervention except to determine the cause of the bradycardia. Bradycardia is often caused by medications. Clients who have multiple chronic diseases are often on multiple medications that can interact with each other. The nurse should assess the clients current medications first.

A client undergoing hemodynamic monitoring after a myocardial infarction has a right atrial pressure of 0.5 mm Hg. What action by the nurse is most appropriate? a. Level the transducer at the phlebostatic axis. b. Lay the client in the supine position. c. Prepare to administer diuretics. d. Prepare to administer a fluid bolus.

ANS: D Normal right atrial pressures are from 1 to 8 mm Hg. Lower pressures usually indicate hypovolemia, so the nurse should prepare to administer a fluid bolus. The transducer should remain leveled at the phlebostatic axis. Positioning may or may not influence readings. Diuretics would be contraindicated.

A nurse is caring for a client with acute pericarditis who reports substernal precordial pain that radiates to the left side of the neck. Which nonpharmacologic comfort measure should the nurse implement? a. Apply an ice pack to the clients chest. b. Provide a neck rub, especially on the left side. c. Allow the client to lie in bed with the lights down. d. Sit the client up with a pillow to lean forward on.

ANS: D Pain from acute pericarditis may worsen when the client lays supine. The nurse should position the client in a comfortable position, which usually is upright and leaning slightly forward. Pain is decreased by using gravity to take pressure off the heart muscle. An ice pack and neck rub will not relieve this pain.

A nursing student is caring for a client with an abdominal aneurysm. What action by the student requires the registered nurse to intervene? a. Assesses the client for back pain b. Auscultates over abdominal bruit c. Measures the abdominal girth d. Palpates the abdomen in four quadrants

ANS: D Abdominal aneurysms should never be palpated as this increases the risk of rupture. The registered nurse should intervene when the student attempts to do this. The other actions are appropriate.

An emergency department nurse triages clients who present with chest discomfort. Which client should the nurse plan to assess first? a. A 42-year-old female who describes her pain as a dull ache with numbness in her fingers b. A 49-year-old male who reports moderate pain that is worse on inspiration c. A 53-year-old female who reports substernal pain that radiates to her abdomen d. A 58-year-old male who describes his pain as intense stabbing that spreads across his chest

ANS: D All clients who have chest pain should be assessed more thoroughly. To determine which client should be seen first, the nurse must understand common differences in pain descriptions. Intense stabbing, vise-like substernal pain that spreads through the clients chest, arms, jaw, back, or neck is indicative of a myocardial infarction. The nurse should plan to see this client first to prevent cardiac cell death. A dull ache with numbness in the fingers is consistent with anxiety. Pain that gets worse with inspiration is usually related to a pleuropulmonary problem. Pain that spreads to the abdomen is often associated with an esophageal-gastric problem, especially when this pain is experienced by a male client. Female clients may experience abdominal discomfort with a myocardial event. Although clients with anxiety, pleuropulmonary, and esophageal-gastric problems should be seen, they are not a higher priority than myocardial infarction.

A nurse assesses a client who has a radial artery catheter. Which assessment should the nurse complete first? a. Amount of pressure in fluid container b. Date of catheter tubing change c. Percent of heparin in infusion container d. Presence of an ulnar pulse

ANS: D An intra-arterial catheter may cause arterial occlusion, which can lead to absent or decreased perfusion to the extremity. Assessment of an ulnar pulse is one way to assess circulation to the arm in which the catheter is located. The nurse would note that there is enough pressure in the fluid container to keep the system flushed, and would check to see whether the catheter tubing needs to be changed. However, these are not assessments of greatest concern. Because of heparin-induced thrombocytopenia, heparin is not used in most institutions for an arterial catheter.

A nurse assesses a client who is scheduled for a cardiac catheterization. Which assessment should the nurse complete prior to this procedure? a. Clients level of anxiety b. Ability to turn self in bed c. Cardiac rhythm and heart rate d. Allergies to iodine-based agents

ANS: D Before the procedure, the nurse should ascertain whether the client has an allergy to iodine-containing preparations, such as seafood or local anesthetics. The contrast medium used during the procedure is iodine based. This allergy can cause a life-threatening reaction, so it is a high priority. Second, it is important for the nurse to assess anxiety, mobility, and baseline cardiac status.

A nurse assesses a client after administering a prescribed beta blocker. Which assessment should the nurse expect to find? a. Blood pressure increased from 98/42 mm Hg to 132/60 mm Hg b. Respiratory rate decreased from 25 breaths/min to 14 breaths/min c. Oxygen saturation increased from 88% to 96% d. Pulse decreased from 100 beats/min to 80 beats/min

ANS: D Beta blockers block the stimulation of beta1-adrenergic receptors. They block the sympathetic (fight-or-flight) response and decrease the heart rate (HR). The beta blocker will decrease HR and blood pressure, increasing ventricular filling time. It usually does not have effects on beta2-adrenergic receptor sites. Cardiac output will drop because of decreased HR.

A nurse teaches pt client who has a history of heart failure. Which statement should the nurse include in this clients discharge teaching? a. Avoid drinking more than 3 quarts of liquids each day. b. Eat six small meals daily instead of three larger meals. c. When you feel short of breath, take an additional diuretic. d. Weigh yourself daily while wearing the same amount of clothing.

ANS: D Clients with heart failure are instructed to weigh themselves daily to detect worsening heart failure to avoid complications. Other signs of worsening heart failure include increasing dyspnea, exercise intolerance, cold symptoms, and nocturia. Fluid overload increases symptoms of heart failure. The client should be taught to eat a heart-healthy diet, balance intake and output to prevent dehydration and overload, and take medications as prescribed. The most important discharge teaching is daily weights as this provides the best data related to fluid retention.

A nurse assesses a client who had an intraosseous catheter placed in the left leg. Which assessment finding is of greatest concern? a. The catheter has been in place for 20 hours. b. The client has poor vascular access in the upper extremities. c. The catheter is placed in the proximal tibia. d. The clients left lower extremity is cool to the touch.

ANS: D Compartment syndrome is a condition in which increased tissue perfusion in a confined anatomic space causes decreased blood flow to the area. A cool extremity can signal the possibility of this syndrome. All other findings are important; however, the possible development of compartment syndrome requires immediate intervention because the client could require amputation of the limb if the nurse does not correctly assess this perfusion problem.

A nurse assesses a client who was started on intraperitoneal therapy 5 days ago. The client reports abdominal pain and feeling warm. For which complication of this therapy should the nurse assess this client? a. Allergic reaction b. Bowel obstruction c. Catheter lumen occlusion d. Infection

ANS: D Fever, abdominal pain, abdominal rigidity, and rebound tenderness may be present in the client who has peritonitis related to intraperitoneal therapy. Peritonitis is preventable by using strict aseptic technique in handling all equipment and infusion supplies. An allergic reaction would occur earlier in the course of treatment. Bowel obstruction and catheter lumen occlusion can occur but would present clinically in different ways.

A nurse teaches a client who is prescribed digoxin (Lanoxin) therapy. Which statement should the nurse include in this clients teaching? a. Avoid taking aspirin or aspirin-containing products. b. Increase your intake of foods that are high in potassium. c. Hold this medication if your pulse rate is below 80 beats/min. d. Do not take this medication within 1 hour of taking an antacid.

ANS: D Gastrointestinal absorption of digoxin is erratic. Many medications, especially antacids, interfere with its absorption. Clients are taught to hold their digoxin for bradycardia. Potassium and aspirin have no impact on digoxin absorption, nor do these statements decrease complications of digoxin therapy.

A nurse assesses a clients peripheral IV site, and notices edema and tenderness above the site. Which action should the nurse take next? a. Apply cold compresses to the IV site. b. Elevate the extremity on a pillow. c. Flush the catheter with normal saline. d. Stop the infusion of intravenous fluids.

ANS: D Infiltration occurs when the needle dislodges partially or completely from the vein. Signs of infiltration include edema and tenderness above the site. The nurse should stop the infusion and remove the catheter. Cold compresses and elevation of the extremity can be done after the catheter is discontinued to increase client comfort. Alternatively, warm compresses may be prescribed per institutional policy and may help speed circulation to the area.

A nurse assesses a clients electrocardiograph tracing and observes that not all QRS complexes are preceded by a P wave. How should the nurse interpret this observation? a. The client has hyperkalemia causing irregular QRS complexes. b. Ventricular tachycardia is overriding the normal atrial rhythm. c. The clients chest leads are not making sufficient contact with the skin. d. Ventricular and atrial depolarizations are initiated from different sites.

ANS: D Normal rhythm shows one P wave preceding each QRS complex, indicating that all depolarization is initiated at the sinoatrial node. QRS complexes without a P wave indicate a different source of initiation of depolarization. This finding on an electrocardiograph tracing is not an indication of hyperkalemia, ventricular tachycardia, or disconnection of leads.

A nurse cares for a client who has advanced cardiac disease and states, I am having trouble sleeping at night. How should the nurse respond? a. I will consult the provider to prescribe a sleep study to determine the problem. b. You become hypoxic while sleeping; oxygen therapy via nasal cannula will help. c. A continuous positive airway pressure, or CPAP, breathing mask will help you breathe at night. d. Use pillows to elevate your head and chest while you are sleeping.

ANS: D The client is experiencing orthopnea (shortness of breath while lying flat). The nurse should teach the client to elevate the head and chest with pillows or sleep in a recliner. A sleep study is not necessary to diagnose this client. Oxygen and CPAP will not help a client with orthopnea.

A nurse delegates care to an unlicensed assistive personnel (UAP). Which statement should the nurse include when delegating hygiene for a client who has a vascular access device? a. Provide a bed bath instead of letting the client take a shower. b. Use sterile technique when changing the dressing. c. Disconnect the intravenous fluid tubing prior to the clients bath. d. Use a plastic bag to cover the extremity with the device.

ANS: D The nurse should ask the UAP to cover the extremity with the vascular access device with a plastic bag or wrap to keep the dressing and site dry. The client may take a shower with a vascular device. The nurse should disconnect IV fluid tubing prior to the bath and change the dressing using sterile technique if necessary. These options are not appropriate to delegate to the UAP.

A nurse assesses a client after administering isosorbide mononitrate (Imdur). The client reports a headache. Which action should the nurse take? a. Initiate oxygen therapy. b. Hold the next dose of Imdur. c. Instruct the client to drink water. d. Administer PRN acetaminophen.

ANS: D The vasodilating effects of isosorbide mononitrate frequently cause clients to have headaches during the initial period of therapy. Clients should be told about this side effect and encouraged to take the medication with food. Some clients obtain relief with mild analgesics, such as acetaminophen. The clients headache is not related to hypoxia or dehydration; therefore, these interventions would not help. The client needs to take the medication as prescribed to prevent angina; the medication should not be held.

A nurse prepares to defibrillate a client who is in ventricular fibrillation. Which priority intervention should the nurse perform prior to defibrillating this client? a. Make sure the defibrillator is set to the synchronous mode. b. Administer 1 mg of intravenous epinephrine. c. Test the equipment by delivering a smaller shock at 100 joules. d. Ensure that everyone is clear of contact with the client and the bed.

ANS: D To avoid injury, the rescuer commands that all personnel clear contact with the client or the bed and ensures their compliance before delivery of the shock. A precordial thump can be delivered when no defibrillator is available. Defibrillation is done in asynchronous mode. Equipment should not be tested before a client is defibrillated because this is an emergency procedure; equipment should be checked on a routine basis. Epinephrine should be administered after defibrillation.


Ensembles d'études connexes

Chapter 1 Foundations of Government

View Set

Alterations in Cardiac Function - Chapter 32

View Set

Laws and Ethics Through Classifications of Laws

View Set